Con đường tăng trưởng cân bằng (Câu hỏi định tính)


5

$ \ textbf {Từ chối trách nhiệm đầy đủ:} $ Tôi đang học cho kỳ thi ứng cử của mình và câu hỏi này là từ một trong những kỳ thi năm ngoái.

$$ max \; \ sum_ {t = 0} ^ \ infty \ beta ^ t \ frac {c_t ^ {1- \ gamma}} {1- \ gamma} $$ Theo các ràng buộc sau:

$$ c_t + i_t = y_t $$   $$ y_t = A_tk_t ^ \ alpha $$   $$ k_ {t + 1} = (1- \ delta) k_t + i_t $$   $$ ln (A_ {t + 1}) = ln (A_t) + \ mu $$   $$ c_t, k_ {t + 1} \ geq 0 $$   (b) Tính toán một lộ trình tăng trưởng cân bằng trong đó vốn, tiêu dùng và sản lượng tăng trưởng với tốc độ không đổi. Trên con đường tăng trưởng cân bằng

(i) Tốc độ tăng trưởng của vốn là gì?

(ii) Tốc độ tăng trưởng của tiêu dùng là gì?

(iii) Tốc độ tăng trưởng của sản lượng là gì?

(iv) Tỷ lệ vốn trên sản lượng là bao nhiêu?

(v) Tỷ lệ tiêu thụ so với đầu ra là bao nhiêu?

$ \ textbf {Công việc của tôi:} $

Kết hợp các ràng buộc chúng ta nhận được: $$ c_t = A_tk_t ^ \ alpha + (1- \ delta) k_t-k_ {t + 1} $$

Lấy hai thuật ngữ từ hàm mục tiêu chúng ta nhận được: $$ ... + \ beta ^ t \ bigg (\ frac {\ big (A_tk_t ^ \ alpha + (1- \ delta k_t-k_ {t + 1} \ big) ^ {1- \ gamma}} {1- \ gamma} \ bigg) + \ beta ^ {t + 1} \ bigg (\ frac {\ big (A_ {t + 1} k_ {t + 1} ^ \ alpha + (1- \ delta k_ {t + 1} -k_ {t + 2} \ lớn) ^ {1- \ gamma}} {1- \ gamma} \ bigg) + ... $$

Phân biệt w.r.t $ k_ {t + 1} $ và tiêu thụ thay thế trở lại trong chúng tôi nhận được: $$ c_ ^ + (1- \ delta) \ bigg] c_ {t + 1} ^ {- \ gamma} $$ $ $$ bigg) + (1- \ delta) \ bigg) \ bigg] ^ {\ frac {1} {\ gamma}} \ qquad (1) $$ Tôi cũng bắt nguồn: $$ \ frac {k_ {t + 1}} {k_t} = (1- \ delta) + \ frac {c_t} {k_t} - \ frac {y_} $$ \ frac {y_ {t + 1}} {y_t} = e ^ \ mu \ bigg (\ frac {k_ {t + 1}} {k_} Chuyến tàu tư tưởng của tôi là như sau:

$ \ frac {k_ {t + 1}} {k_t} = \ frac {c_ {t + 1}} {c_t} $ vì nếu $ \ frac {k_ {t + 1}} {k_t} & gt; \ frac { c_ {t + 1}} {c_t} $ chúng tôi sẽ không thỏa mãn điều kiện chuyển đổi vì chúng tôi sẽ có vốn trong giai đoạn "cuối cùng", điều đó có nghĩa là chúng tôi không thể tối đa hóa mức tiêu thụ. Nếu $ \ frac {k_ {t + 1}} {k_t} & lt; \ frac {c_ {t + 1}} {c_t} $, cuối cùng chúng ta sẽ không thể duy trì mức tiêu thụ không đổi. Sử dụng điều đó và thực tế là tỷ lệ đầu ra so với vốn sẽ không đổi theo thời gian, việc cân bằng các phương trình (1) và (2) mang lại kết quả lộn xộn sau: $$ \ bigg (\ frac {y_t} {k_t} \ bigg) ^ \ gamma + \ alpha \ beta \ bigg (\ frac {y_t} {k_t} \ bigg) = \ bigg ((1- \)) {c_t} {k_t} \ bigg) ^ \ gamma + \ beta (1- \ delta) $$ Theo như tôi có thể nói, phương trình này không thể giải được. Điều này có nghĩa là tôi không thể nói rằng tốc độ tăng trưởng của tiêu dùng bằng với tốc độ tăng trưởng của vốn hay tôi đã phạm sai lầm ở đâu đó trên đường đi? Ngoài ra, làm thế nào một người sẽ giải quyết cho tốc độ tăng trưởng nếu không theo cách này ??


Bạn dường như bỏ qua "gợi ý" được cung cấp bởi phương trình $ \ ln (A_ {t + 1}) $, (bạn không nghĩ rằng bạn nên xem xét tại sao nó được đưa ra trong logarit?) chỉ trích thành phần mang lại sự tăng trưởng liên tục (nếu không, sản phẩm cận biên giảm dần của vốn sẽ dẫn đến tăng trưởng bằng không). Cũng nhớ rằng $ \ ln x_ {t + 1} - \ ln x_ {t} $ là một xấp xỉ chấp nhận được với tốc độ tăng trưởng của một biến. Sử dụng hai điều này để tuyến tính hóa mọi thứ (và khi bạn làm, bạn có thể xem xét đăng câu trả lời cho câu hỏi của riêng bạn, đây là cách thực hành hoàn toàn chấp nhận được).
Alecos Papadopoulos

@AlecosPapadopoulos cảm ơn tôi sẽ xem!
DornerA

@AlecosPapadopoulos Tôi vẫn có thể khẳng định tốc độ tăng trưởng của vốn bằng với tốc độ tăng trưởng cho tiêu dùng? Hay điều này là không cần thiết?
DornerA

Đó là, và vì những lý do bạn đã giải thích trong câu hỏi của bạn.
Alecos Papadopoulos

@AlecosPapadopoulos Vì vậy, tôi hiểu những gì bạn đã nói với tôi rằng hai điều đó dẫn đến $ ln (y_ {t + 1}) - ln (y_t) = \ mu + \ alpha (ln (k_ {t + 1}) - ln (k_t )) $ và cho phép đầu ra ở trên mức 0, nhưng tôi vẫn bị kẹt ở đó. Tôi không thể tìm ra cách để có một phương trình giải một trong các biến của mình như $ \ frac {y_t} {k_t} $. Tôi không biết có quá nhiều để hỏi không, nhưng bạn có thể cho tôi biết ý tưởng về bất kỳ thao tác phương trình nào mà tôi có thể cần phải làm không? Tôi đã cố gắng để xem nó sẽ trông như thế nào nếu tôi tuyến tính hóa tất cả các ràng buộc, nhưng điều đó dường như không có ích vì tôi không thể tách ra bất kỳ thuật ngữ nào được thêm vào.
DornerA

Câu trả lời:


1

. tốc độ tăng trưởng không đổi trong thời gian liên tục).

Từ $$ \ frac {y_ {t + 1}} {y_t} = e ^ \ m

Lấy nhật ký bạn nhận được

$$ g_y = \ mu + \ alpha g_k $$

Trong đó "$ g $" là tốc độ tăng trưởng. Một trong những con đường tăng trưởng cân bằng, chúng phải bằng một số hằng số, giả sử $ g ^ * $, vì vậy bạn có

$ $$

đó là tốc độ tăng trưởng của nền kinh tế trên con đường tăng trưởng cân bằng.

Bạn cũng có thể (bằng cách mô phỏng thao tác phương trình mà bạn bắt đầu hiển thị công việc của mình), để dễ dàng đến

$ $$ Vì vậy, bạn chỉ cần xác định một trong hai để có được cái kia.

Nhận ra cách bạn có thể viết phía bên trái của $ (1) $ trong khi trên đường tăng trưởng cân bằng, sau đó xoay phương trình và giải quyết cho $ k / y $. Đừng mong đợi để có được một cái gì đó nhất thiết phải "nhìn đơn giản".

Khi sử dụng trang web của chúng tôi, bạn xác nhận rằng bạn đã đọc và hiểu Chính sách cookieChính sách bảo mật của chúng tôi.
Licensed under cc by-sa 3.0 with attribution required.